LSAT and Law School Admissions Forum

Get expert LSAT preparation and law school admissions advice from PowerScore Test Preparation.

 Administrator
PowerScore Staff
  • PowerScore Staff
  • Posts: 8919
  • Joined: Feb 02, 2011
|
#32014
Complete Question Explanation
(The complete setup for this game can be found here: lsat/viewtopic.php?t=15241)

The correct answer choice is (D)

This Global, Cannot Be True question tests an inference that you may not have: which ranger is never assigned to area 3? If you don’t know the answer, don’t worry. Simply skip this question for now, and tackle some of the local questions in this game. The more solutions you accumulate, the more evidence you’ll have as to which rangers could be assigned to area 3. This, in turn, will allow you to eliminate at least some—and potentially all!—of the incorrect answer choices. In fact, we can eliminate answer choices (A), (B), and (C) already using this method. The only answer choices under consideration should be (D) and (E).

Of course, as we will see, since L requires pairing with either K or M, but never with both, then placing K and M together in 3 is immediately problematic: L going in 3 with them breaks the third rule, and L going in either 1 or 2 without them also breaks rule 3.

Similarly, if you approached this game using the two templates as shown previously, it would be immediately clear that K cannot be assigned to area 3. In Template 1, K is assigned to area 1, whereas in Template 2 we have a K Not Law for area 3. Thus, K is never assigned to area 3.

Answer choice (A): This answer choice is incorrect, because P can be assigned to area 3. See Question #7.

Answer choice (B): This answer choice is incorrect, because O can be assigned to area 3. See Template 2, as well as Question #8.

Answer choice (C): This answer choice is incorrect, because L can be assigned to area 3. See Question #8.

Answer choice (D): This is the correct answer choice. Many test takers arrive at this answer choice by the process of elimination, having proven that P, O, L, and J can each be assigned to area 3. However, if you determined that K cannot be assigned to area 3 because of L’s need for either a K or M partnership (or because neither template allows for such an outcome) then you need not waste any time eliminating the four remaining contenders. Just pick answer choice (D) and move on.

Answer choice (E): This answer choice is incorrect, because J could be assigned to area 3. This is the only answer choice that cannot easily be eliminated by reference to some of the local setups in this game, so it deserves a closer look.

Interestingly, in attempting to test whether J can be assigned to area 3, we must first recognize that O cannot be in 2: O in 2 would force J and K to be together, meaning putting them both in area 3 for this hypothetical, which closes the group and causes an immediate issue L as it cannot go with either K or M. So it’s possible that by the very first step in our testing we would see the problem with K and M in the same group, and thus solve this question straight away.

If not, then we would at least recognize that O must be in 3 for J to be in 3, and thus we need to work with Template 2 (recall that J is assigned to area 1 in Template 1):
PT79_Game_#2_#9_diagram 1.png
Having reached the maximum number of rangers in area 3, we now must assign P to area 2, so as to ensure that P is not assigned to area 1 (second rule). The two remaining variables—K and L—must be assigned to the same area as each other in compliance with the third rule. And, since each area must have at least one ranger assigned to it, it follows that K and L must both be assigned to area 1:
PT79_Game_#2_#9_diagram 2.png
This solution does not violate any of the rules, showing that J could be assigned to area 3.

Get the most out of your LSAT Prep Plus subscription.

Analyze and track your performance with our Testing and Analytics Package.